Autor Tema: Ejercicio. Proyectores

0 Usuarios y 1 Visitante están viendo este tema.

23 Febrero, 2021, 03:13 am
Leído 1359 veces

Mimi-chan

  • $$\Large \color{#6a84c0}\pi$$
  • Mensajes: 11
  • País: ar
  • Karma: +0/-0
  • Sexo: Femenino
Sean \( E_{1},E_{2},...,E_{k}\in L(V) \) con V un \( \mathbb{C} \) espacio vectorial, tales que \( E_{1}+E_{2}+...+E_{k}=I \)
a)Probar que si \(  E_{i}E_{j}=0  \) siempre que \( i\neq j \), entonces \(  E_{i}^{2}=E_{i}\forall{i} \)
b)Probar el reciproco del item anterior.
Creo que consegui demostrar el inciso a, pero no se como realizar el b
Resolucion a:
\( E_{i}(E_{1}+E_{2}+...+E_{i})=E_{i}(I)  \)
\( E_{i}E_{1}+E_{i}E_{2}+...+E_{i}^{2}=E_{i} \) por hipotesis \( E_{i}E_{1}=E_{i}E_{2}=E_{i}E_{j}=0  \)
\( E_{i}^{2}=E_{i} \)

Para el b entiendo que : Si \( E_{1}+E_{2}+...+E_{k}=I \) y \(  E_{i}^{2}=E_{i} \), entonces \( E_{i}E_{j}=0  \) pero no se como empezar.

24 Febrero, 2021, 11:00 am
Respuesta #1

geómetracat

  • Moderador Global
  • Mensajes: 3,924
  • País: es
  • Karma: +0/-0
  • Sexo: Masculino
La parte a) está bien, aunque escrita de manera un poco rara (si escribes \[ E_1+\dots+E_i \] yo entiendo que hay \[ i \] sumandos y no \[ k \] como debería haber).

El apartado b) es más complicado (o por lo menos yo no he sido capaz de ver una forma muy fácil de hacerlo). Puedes hacerlo de la siguiente manera. Como para cada \[ i \] tienes \[ E_i^2=E_i \], los \[ E_i \] son proyectores y tienes \[ V=im(E_i)\oplus ker(E_i) \].

Por otro lado, del hecho de que se cumple \[ E_1+\dots + E_k=I \] se tiene que \[ V=im(E_1)\oplus im(E_2) \dots \oplus im(E_k) \].
La justificación en spoiler está mal:
Spoiler
En efecto, que \[ V \] es suma de las imágenes es inmediato de \[ E_1+\dots + E_k=I \], y para ver que la suma es directa basta ver que \[ im(E_i) \cap \sum_{j \neq i} im(E_j)=0 \] para cada \[ i \]. Pero si tienes \[ v \in im(E_i) \cap \sum_{j \neq i} im(E_j) \], puedes escribir \[ v=\sum_{j \neq i} v_j \] donde cada \[ v_j \in im(E_j) \]. Por tanto \[ E_jv=v_j \] para todo \[ j \neq i \]. Por otro lado, como \[ v \in im(E_i) \], tienes que \[ E_i v=v \]. Aplicando ambos lados de \[ E_1+\dots + E_k=I \] a \[ v \] obtienes pues \[ 2v=v \] de donde \[ v=0 \], como queríamos.
[cerrar]
Para ver esto, que \[ V \] es suma de las imágenes es inmediato de \[ E_1+\dots + E_k=I \]. Para ver que la suma es directa, vamos a ver que \[ \dim(im(E_1))+\dots + \dim(im(E_k))=\dim(V) \]. Para esto usamos que si \[ E \] es un proyector, se tiene que \[ \dim(im(E))=tr(E) \], donde \[ tr(E) \] es la traza (para ver esto, como \[ E^2=E \], el polinomio mínimo de \[ E \] es \[ x(x-1) \], con lo que \[ E \] es diagonalizable con valores propios \[ 0 \] y \[ 1 \]). Ahora, como \[ E_1+\dots+E_k=I \], \[ tr(E_1)+\dots + tr(E_k)=tr(I)=n \], de donde \[ \dim(im(E_1))+\dots + \dim(im(E_k))=\dim(V) \].


Y ahora que sabemos \[ V=im(E_1) \oplus \dots \oplus im(E_k) \] y que para cada \[ i \] se tiene \( V=im(E_i) \oplus ker(E_i) \) ya está, pues esto implica que si \[ i \neq j \] tienes \[ im(E_j) \subset ker(E_i) \] (para ver esto ten en cuenta que \[ E_i v \] es la componente de \[ v \] en \[ im(E_i) \] en ambas descomposiciones de \[ V \]), luego \[ E_iE_j=0 \].
La ecuación más bonita de las matemáticas: \( d^2=0 \)

24 Febrero, 2021, 11:09 am
Respuesta #2

Luis Fuentes

  • el_manco
  • Administrador
  • Mensajes: 56,002
  • País: es
  • Karma: +0/-0
Hola

El apartado b) es más complicado (o por lo menos yo no he sido capaz de ver una forma muy fácil de hacerlo). Puedes hacerlo de la siguiente manera. Como para cada \[ i \] tienes \[ E_i^2=E_i \], los \[ E_i \] son proyectores y tienes \[ V=im(E_i)\oplus ker(E_i) \].
Por otro lado, del hecho de que se cumple \[ E_1+\dots + E_k=I \] se tiene que \[ V=im(E_1)\oplus im(E_2) \dots \oplus im(E_k) \]. En efecto, que \[ V \] es suma de las imágenes es inmediato de \[ E_1+\dots + E_k=I \], y para ver que la suma es directa basta ver que \[ im(E_i) \cap \sum_{j \neq i} im(E_j)=0 \] para cada \[ i \]. Pero si tienes \[ v \in im(E_i) \cap \sum_{j \neq i} im(E_j) \], puedes escribir \[ v=\sum_{j \neq i} v_j \] donde cada \[ v_j \in im(E_j) \]. Por tanto \[ E_jv=v_j \] para todo \[ j \neq i \].

¿Lo qué he marcado en rojo es obvio? No lo veo ahora mismo.

Saludos.

24 Febrero, 2021, 11:20 am
Respuesta #3

geómetracat

  • Moderador Global
  • Mensajes: 3,924
  • País: es
  • Karma: +0/-0
  • Sexo: Masculino
No lo es, me he colado ahí. Tengo que pensarlo mejor.
La ecuación más bonita de las matemáticas: \( d^2=0 \)

24 Febrero, 2021, 12:08 pm
Respuesta #4

geómetracat

  • Moderador Global
  • Mensajes: 3,924
  • País: es
  • Karma: +0/-0
  • Sexo: Masculino
He encontrado por ahí la pieza que faltaba para ver que \[ V \] es suma directa de las imágenes. Creo que ahora está bien.
La ecuación más bonita de las matemáticas: \( d^2=0 \)

24 Febrero, 2021, 12:21 pm
Respuesta #5

Luis Fuentes

  • el_manco
  • Administrador
  • Mensajes: 56,002
  • País: es
  • Karma: +0/-0
Hola

Para ver esto, que \[ V \] es suma de las imágenes es inmediato de \[ E_1+\dots + E_k=I \]. Para ver que la suma es directa, vamos a ver que \[ \dim(im(E_1))+\dots + \dim(im(E_k))=\dim(V) \]. Para esto usamos que si \[ E \] es un proyector, se tiene que \[ \dim(im(E))=tr(E) \], donde \[ tr(E) \] es la traza (para ver esto, como \[ E^2=E \], el polinomio mínimo de \[ E \] es \[ x(x-1) \], con lo que \[ E \] es diagonalizable con valores propios \[ 0 \] y \[ 1 \]). Ahora, como \[ E_1+\dots+E_k=I \], \[ tr(E_1)+\dots + tr(E_k)=tr(I)=n \], de donde \[ \dim(im(E_1))+\dots + \dim(im(E_k))=\dim(V) \].


 :aplauso: Pues creo que ahora está bien. La verdad es que a mi se me escurría por algún detalle esta demostración, cada vez que la intenté.

Con el truquillo de traza igual a dimensión, la verdad es que queda muy elegante y extremadamente sencillo.

Citar
Y ahora que sabemos \[ V=im(E_1) \oplus \dots \oplus im(E_k) \] y que para cada \[ i \] se tiene \( V=im(E_i) \oplus ker(E_i) \) ya está, pues esto implica que si \[ i \neq j \] tienes \[ im(E_j) \subset ker(E_i) \] (para ver esto ten en cuenta que \[ E_i v \] es la componente de \[ v \] en \[ im(E_i) \] en ambas descomposiciones de \[ V \]), luego \[ E_iE_j=0 \].

Para concluir a mi me sale mas natural simplemente teniendo en cuenta que:

\( E_1v=E_1E_1v+E_2E_1v+\ldots E_kvE_1v=E_1v+E_2E_1v+\ldots E_kE_1v \)

y como la suma de imágenes es suma directa, por la unicidad de la descomposición todos los \( E_iE_1v=0 \) para \( i>1 \).

Lo análogo si cambiamos \( E_1 \) por los otros operadores.

Como cabo suelto ahí usamos que el espacio vectorial es de dimensión finita. ¿Y en dimensión infinita?.

Saludos.

24 Febrero, 2021, 02:14 pm
Respuesta #6

geómetracat

  • Moderador Global
  • Mensajes: 3,924
  • País: es
  • Karma: +0/-0
  • Sexo: Masculino
Para concluir a mi me sale mas natural simplemente teniendo en cuenta que:

\( E_1v=E_1E_1v+E_2E_1v+\ldots E_kvE_1v=E_1v+E_2E_1v+\ldots E_kE_1v \)

y como la suma de imágenes es suma directa, por la unicidad de la descomposición todos los \( E_iE_1v=0 \) para \( i>1 \).

Lo análogo si cambiamos \( E_1 \) por los otros operadores.
Mucho mejor así, desde luego.

Citar
Como cabo suelto ahí usamos que el espacio vectorial es de dimensión finita. ¿Y en dimensión infinita?.
Buena pregunta. La verdad es que no tengo ni idea, pero con esta generalidad no me extrañaría que fuera falso.
La ecuación más bonita de las matemáticas: \( d^2=0 \)

25 Febrero, 2021, 10:45 am
Respuesta #7

Luis Fuentes

  • el_manco
  • Administrador
  • Mensajes: 56,002
  • País: es
  • Karma: +0/-0
Hola

 Pues efectivamente en el caso infinito es falso. Encontré un ejemplo por ahí. Lo transcribo:

 Si consideramos el espacio vectorial \( V \) de funciones \( f:\Bbb Z\to \Bbb R \) con soporte finito, tenemos la base canónica:

\( f_n:\Bbb Z\to \Bbb R,\quad f_n(m)=\begin{cases}{1}&\text{si}& n=m\\0 & \text{si}& n\neq m\end{cases} \)

 Si definimos:

\(  E_1(f_n)=\dfrac{n}{2}(f_n+f_{2-n}) \)

\(  E_2(f_n)=\dfrac{n}{2}(f_n-f_{2-n}) \)

\(  E_3(f_n)=-\dfrac{n}{2}(f_n+f_{-2-n}) \)

\(  E_4(f_n)=-\dfrac{n}{2}(f_n-f_{-2-n}) \)
\( E_5(f_n)=f_n \)

 es fácil ver que \( E_i^2=E_i \), \( E_1+E_2+E_3+E_4+E_5=Id \), pero claramente \( E_5E_i\neq 0 \) para \( i=1,2,3,4 \).

Saludos.

P.D. Si no me equivoco para \( k=2,3,4 \) el resultado si es cierto para dimensión infinita. Si puedo luego intento escribir una prueba. Si es así esto si cerraría por completo la cuestión.

25 Febrero, 2021, 11:06 am
Respuesta #8

Luis Fuentes

  • el_manco
  • Administrador
  • Mensajes: 56,002
  • País: es
  • Karma: +0/-0
Hola

P.D. Si no me equivoco para \( k=2,3,4 \) el resultado si es cierto para dimensión infinita. Si puedo luego intento escribir una prueba. Si es así esto si cerraría por completo la cuestión.

Para \( k=2 \) es inmediato. Si \( E_1+E_2=Id \), \( E_1^2+E_2E_1=E_1 \) y por tanto \( E_2E_1=0 \).

Para \( k=3 \). Si \( E_1+E_2+E_3=Id \) entonces \( E_1+E_2=Id-E_3 \) y "elevando al cuadrado" (componiendo consigo mismo cada término):

\( E_1^2+E_1E_2+E_2E_1+E_2^2=Id-2E_3+E_3^2=Id-E_3=E_1+E_2 \)

De donde \( E_1E_2+E_2E_1=0 \). Dado cualquier vector \( u \):

\( E_1E_2u=E_1E_2E_2u=-E_2E_1(E_2u)\quad \Rightarrow{}\quad E_2(E_1E_2u)=-E_1E_2u \)

Por tanto \( E_1E_2u \) sería un autovector de \( E_2 \) asociado al \( -1 \). Como \( E_2 \) es idempotente necesariamente \( E_1E_2u=0 \).

Queda el caso \( k=4 \).

Saludos.

22 Octubre, 2021, 05:50 pm
Respuesta #9

Dark

  • $$\Large \color{#5e8d56}\pi\,\pi\,\pi$$
  • Mensajes: 426
  • País: 00
  • Karma: +0/-0
  • Sexo: Masculino
Me surge una duda, para ver que $$tr(E)=dim(Im E)$$, Como $$E$$ es una proyección entonces $$E(E-I)=0$$ entonces como los autovalores solo pueden ser 0 y 1. Por lo tanto, $$p(x)=x(x-1)$$ es el polinomio mínimo de $$E$$, pero qué quiere decir que sea el polinomio mínimo? no manejo ese concepto, aunque parece que va relacionado con el polinomio característico.

Entonces que los valores propios sean 0 y 1, es decir distintos implicaría que $$E$$ es diagonalizable. Pero no logro relacionar la $$tr(E)=\sum_{}e_ie_i$$ y $$rang(E)$$, acaso $$E$$ es una matriz que solo posee unos en la diagonal y ceros fuera de ella?